A baker used 12 cups of batter to make muffins. It took 2/3 of a cup of batter to make one muffin. How many did the baker make?

Answers

Answer 1

Answer:

8 cups

Step-by-step explanation:

1 cup = 2/3

so,2/3 x 12 = 8 cups

Answer 2

Answer:

8 cups

Step-by-step explanation:


Related Questions

Assume that the radius of the hydrogen nucleus is 1.4 · 10-15 meters. How much larger than the nucleus is the entire hydrogen atom? (Calculate the atomic radius for n = 1. Round answer to nearest tenth.)

________times larger than the nucleus.

(A). 3.8 x 10⁴
(B). 3.8 x 10¹⁴
(C). 3.8 x 10^-5

(15 points reward)

Answers

Answer:

A

Step-by-step explanation:

I did not look up the actual numbers, but it can only be A.

of course, the whole aim is larger than the nucleus, which is why C is impossible with its negative exponent (which would make the whole aim smaller than the nucleus).

and B. can't be true, because it is so big 10¹⁴ times bigger than a 10-¹⁵ atom ? this would make the whole atom the size of about 10-¹ meters. so, 10 cm. a single hydrogen atom would be bigger than a tennis ball. which it isn't.

so, that only leaves A.

find the equation of the median from b in ABC whose vertices are (1,5), B(5,3) and C(-3, -2)

Answers

Answer:

y = x + 6

x = 1

y = ¼(x - 5) + 3

Step-by-step explanation:

Vetices are;

A(1,5), B(5,3) and C(-3, -2)

Thus;

Median of AB is; D = (1 + 5)/2, (5 + 3)/2

D = (3, 4)

Median of BC is; E = (5 + (-3))/2, (3 + (-2))/2

E = (1, 0.5)

Median of AC is; F ; (-3 + 1)/2, (-2 + 5)/2

F = (-1, 1.5)

Thus, the median lines will be;

CD, AE & BF.

Thus;

Equation of CD is;

(y - (-3))/(x - (-2)) = (-2 - 4))/(-3 - 3)

(y + 4)/(x + 2) = -6/-6

y - 4 = 1(x + 2)

y = 4 + x + 2

y = x + 6

Equation of AE;

(y - 5)/(x - 1) = (0.5 - 5)/(1 - 1)

(y - 5)/(x - 1) = -4.5/0

Cross multiply to get;

0(y - 5) = -4.5(x - 1)

-4.5x = -4.5

x = 1

Equation of BF;

(y - 3)/(x - 5) = (1.5 - 3)/(-1 - 5)

(y - 3)/(x - 5) = -1.5/-6

(y - 3)/(x - 5) = 1/4

y - 3 = ¼(x - 5)

y = ¼(x - 5) + 3

Expand and simplify
3(4m - 3t)-2(m – 2t)

Answers

Answer:

10m-5t

Step-by-step explanation:

3(4m-3t) - 2(m-2t)

(12m - 9t) + (-2m+4t)

12m-2m - 9t+4t

10m-5t

Answer:

Step-by-step explanation:

3(4m - 3t) = 12m - 9t    

-2(m - 2t) = -2m + 4t

Write the 2 terms together.

12m - 9t -2m + 4x

12m - 2m - 9t + 4t

10m - 5t

2. Given: HM = VM, and ZH and cV Triangle congruence criteria,
are right angles.
Is AGHM = AUVM?
sss
SAS
ASA
AAS
HL
G
I
M
V
U

Answers

Answer: SAS is the correct criteria

Explanation:

Angles VMU and GMH are congruent by the Vertical Angles Theorem. Given that angles UVM and GHM are congruent because they are both right angles, we now have two pairs of corresponding angles. Also given that sides HM and VM are congruent, we now have two corresponding pairs of congruent angles and a pair of congruent sides.Therefore, your best option is the ASA postulate, which states that if two angles and the included side of one triangle are congruent to two angles and the included side of another triangle, then the two triangles are congruent. Therefore, we have a corresponding angle, a corresponding side, and another corresponding angle in triangle GHM, which is congruent to its corresponding angle, a corresponding side, and another corresponding angle in triangle UVM.

Which rate is equivalent to $800 per 40 hours?

Answers

$800 per 40 hours can be expressed as a fraction as 800/40. We can find the unit rate by dividing the numerator and denominator by 40. 800/40 is 20, and 40/40 is 1. This means that the unit rate is $20 per hour.

this is a geometry question, i need something quickly :)

Answers

Answer:

hope it helps mark me brainlieast!

Step-by-step explanation:

For triangle ABC with sides  a,b,c  labeled in the usual way,

c2=a2+b2−2abcosC  

We can easily solve for angle  C .

2abcosC=a2+b2−c2  

cosC=a2+b2−c22ab  

C=arccosa2+b2−c22ab  

That’s the formula for getting the angle of a triangle from its sides.

The Law of Cosines has no exceptions and ambiguities, unlike many other trig formulas. Each possible value for a cosine maps uniquely to a triangle angle, and vice versa, a true bijection between cosines and triangle angles. Increasing cosines corresponds to smaller angles.

−1≤cosC≤1  

0∘≤C≤180∘  

We needed to include the degenerate triangle angles,  0∘  and  180∘,  among the triangle angles to capture the full range of the cosine. Degenerate triangles aren’t triangles, but they do correspond to a valid configuration of three points, namely three collinear points.

The Law of Cosines, together with  sin2θ+cos2θ=1 , is all we need to derive most of trigonometry.  C=90∘  gives the Pythagorean Theorem;  C=0  and  C=180∘  give the foundational but often unnamed Segment Addition Theorem, and the Law of Sines is in there as well, which I’ll leave for you to find, just a few steps from  cosC=  … above. (Hint: the Law of Cosines applies to all three angles in a triangle.)

The Triangle Angle Sum Theorem,  A+B+C=180∘ , is a bit hard to tease out. Substituting the Law of Sines into the Law of Cosines we get the very cool

2sinAsinBcosC=sin2A+sin2B−sin2C  

Showing that’s the same as  A+B+C=180∘  is a challenge I’ll leave for you.

In Rational Trigonometry instead of angle we use spreads, squared sines, and the squared form of the formula we just found is the Triple Spread Formula,

4sin2Asin2B(1−sin2C)=(sin2A+sin2B−sin2C)2  

true precisely when  ±A±B±C=180∘k , integer  k,  for some  k  and combination of signs.

This is written in RT in an inverted notation, for triangle  abc  with vertices little  a,b,c  which we conflate with spreads  a,b,c,  

(a+b−c)2=4ab(1−c)  

Very tidy. It’s an often challenging third degree equation to find the spreads corresponding to angles that add to  180∘  or zero, but it’s a whole lot cleaner than the trip through the transcendental tunnel and back, which almost inevitably forces approximation.

The fence posts need to be painted Each post is round with a diameter of 50mm and length of 2.5m. The posts are hollow (no top or bottom) Now work out the surface area of a single fence post

Answers

Answer:

0.3927 m²

Step-by-step explanation:

Since a post is open at both ends, then it means it has no top nor bottom and as such the surface area is;

S.A = 2πrh

Where;

r is radius

h is height

We are given;

diameter; d = 50mm = 0.05 m

We know that; radius = diameter/2 = 0.05/2 = 0.025 m

Height; h = 2.5 m

Thus;

S.A = 2 × π × 0.025 × 2.5

S.A = 0.3927 m²

PLS HELP!!!!
Determine the value of x.
A) 160°
B) 78°
C) 240°
D) 80°

Answers

Answer:

D) 80°

Step-by-step explanation:

(n - 2)180 = 4(180) = 720

78 + 134 + 136 + 132 + 2x + x = 720

3x + 480 = 720

3x = 240

x = 80

Answer:

The answer is D:80 degrees.

Step-by-step explanation:

In the given figure AOB is a straight line find the value of x.
Please tell the correct answer..​

Answers

Answer:

2x+5x+3x=180°

10x=180°

x=180/10

x=18° ans..

Answer:

18

Step-by-step explanation:

Sum of angles in a straight line is 180,

2x + 5x + 3x = 180

10x = 180

x = 180 / 10

x = 18

State if the triangles are similar. If so, how do you know they are similar and complete the similarity statement.
ΔTUV
~ ____

Answers

Answer:

YES by SAS

triangle TUV is similar to triangle TLM

Step-by-step explanation:

The ratios of the sides

24/6 = 4

36/9 = 4

so the sides are similar

We know angle UTV = angle LTM because they are vertical angles

we have side included angle side

The triangles are similar by SAS  ( side angle side)

triangle TUV is similar to triangle TLM

Students taking an online english test are randomly assigned 3 questions out of a set of 9 different questions. How many different sets of questions (i.e. tests) are possible for the students?

Answers

Answer:

2 sets I think

Step-by-step explanation:

what are the answer choices

A 3^{\text{rd}}3 rd 3, start superscript, start text, r, d, end text, end superscript degree binomial with a constant term of 88

Answers

Answer:

See explanation

Step-by-step explanation:

A 3rd degree binomial with a constant term of 8

A binomial expression is an expression which has only terms such as: x² + 5

The degree of a polynomial is the term with the highest exponent on its variable.

Example: the expression above x² + 5

The exponent of variable, x is 2

So, it is a 2nd degree polynomial

We also have 1st degree polynomial where the highest exponent on the variable is 1

3rd degree polynomial where the highest exponent on the variable is 3

A 3rd degree binomial with a constant term of 8

1. There must be a variable, let say x

2. The highest exponent on the variable must be 3

3. There must be a constant 8

4. The expression must have two terms only

It could be x² + 8 where the coefficient of x is 1

2x² + 8

3x² + 8

It could take any form as long as the highest exponent on the variable is 3 and there are just two terms

Answer:

-5x^2+88

Step-by-step explanation:

Parallelogram PARL is similar to parallelogram WXYZ. If AP = 18, PL = 24, and WZ = 96, find the value of c.
A. 4
B.96
C.42
D.72
somebody can help me

Answers

Answer: c = 72

Step-by-step explanation:

You didn't tell us which segment has a length of c, but I'm assuming you meant WX because it corresponds to PA. If two figures are similar, we know that their side length are in proportion. With this, we can set up our proportion[tex]\frac{18}{24} =\frac{c}{96}[/tex] where c is the length of WX. By cross multiplying and dividing, you get  72 for the value of c.

25)
Jackson's current salary is $36,000 per year. Each year his salary is 1.04 times the previous yeal's salary. What
will his salary be in his 5th year?
OA) $42,214.92
OB) $42,114.91

Answers

Answer:

$43,799.50

Step-by-step explanation:

USing the formula:

A = P(1+r)ⁿ

n is the time = 5

1 + r = 1.04

P = 36,000

Substitute the values into the formula

A = 36000(1.04)⁵

A = 36,000(1.2166529024)

A = 43,799.50

Hence the value in the fifth year will e $43,799.50

Can you answer this math homework? Please!

Answers

Answer:

1. one solution (2.5, 0)

2. D. x-5/2y = 25/2

3. No solution

4. (0,2)

5. y = 4x + 5

Step-by-step explanation:

Simplify the expression.
4 * 22 ÷ 2 + 2

Answers

Answer:

46

Step-by-step explanation:

4 * 22 ÷ 2 + 2

Multiply and divide from left to right

88 ÷ 2 + 2

44 +2

Add

46

[tex]\huge\textsf{Hey there!}[/tex]

[tex]\mathsf{\dfrac{4\times22}{2}+2}[/tex]

[tex]\mathsf{4\times22 = \bf 88}[/tex]

[tex]\mathsf{\dfrac{88}{2}+2}[/tex]

[tex]\mathsf{\dfrac{88}{2}=\bf 44}[/tex]

[tex]\mathsf{= 44 + 2}[/tex]

[tex]\mathsf{\bf = 46}[/tex]

[tex]\huge\checkmark\boxed{\huge\textsf{Possible answer \#1: \bf 46}}\huge\checkmark[/tex]

[tex]\huge\text{OR........}[/tex]

[tex]\mathsf{\dfrac{4\times2^2}{2}+2}[/tex]

[tex]\mathsf{2^2}[/tex]

[tex]\mathsf{= 2\times2}[/tex]

[tex]\mathsf{= \bf 4}[/tex]

[tex]\mathsf{= \dfrac{4\times4}{2}+2}[/tex]

[tex]\mathsf{4\times4}[/tex]

[tex]\mathsf{\bf = 16}[/tex]

[tex]\mathsf{= \dfrac{16}{2}+2}[/tex]

[tex]\mathsf{\dfrac{16}{2}}[/tex]

[tex]\mathsf{= \bf 8}[/tex]

[tex]\mathsf{= 8 + 2}[/tex]

[tex]\mathsf{\bf = 10}[/tex]

[tex]\huge\checkmark\boxed{\huge\textsf{Possible answer \#2: \bf 10}}\huge\checkmark[/tex]

[tex]\large\textsf{Good luck on your assignment and enjoy your day!}[/tex]

~[tex]\frak{Amphitrite1040:)}[/tex]

Find the value of a.
A. 16
B. 38.5
C. 15
D. 10

Answers

Answer:

Step-by-step explanation:

divide 60/31 55/46 the answer you multiply

Calculate the mean, median, and mode of the following set of data. Round to the nearest tenth. 12,7,12,9,2,15,6,6,6,1,11

Answers

Answer:

Mean: 7.9
Median: 7
Mode: 6

Step-by-step explanation:

Sort the date set!

1, 2, 6, 6, 6, 7, 9, 11, 12, 12, 15

Then start by finding the mean of the data set!

Mean: the sum of all numbers divided by the amount of numbers

Mean = 1 + 2 + 6 + 6 + 6 + 7 + 9 + 11 + 12 + 12 + 15 = 87 divided by 11 = 7.9

Median: middle number in the data set

Data set: 1, 2, 6, 6, 6, 7, 9, 11, 12, 12, 15

Median = 7 because it is in the middle

Mode: the number that is being used the most

Data set: 1, 2, 6, 6, 6, 7, 9, 11, 12, 12, 15

Mode = 6 because it is used 3 times



Can someone help me with this math homework please!

Answers

Answer:

It's 2, 1, and y = 2x + 1.

Step-by-step explanation:

You can see the rise is 2 and the run is 1, making the slope = 2, and the y-intercept is 1 because that is where it crosses the y axis. Once you have the slope and y intercept, you can put it in a function, with the form being y=2x+1, the slope being the number before the x and the y-int value being after the x.

Answer:

1.  2

2. 1

3. y = 2x+1

Step-by-step explanation:

1. [tex]\frac{rise}{run}[/tex]

2. Where does the line cross the y-axis?

3. y = mx+b

m= slope

b = y-intercept

Abel bought a mini hi-fi set for S600.
He sold it to Bob at a loss of 20%.
Bob sold it to Charles and made a profit of 5%. How much did Charles pay for it?

Answers

Answer:

$504

$600* .8 = $480

$480 * 1.05 = $504

Step-by-step explanation:

Answer:

Step-by-step explanation:

Abel:

Cost price = $ 600

Loss = 20%

Selling price = [tex]\frac{100-loss}{100}*Cost \ price[/tex]

                     [tex]= \frac{(100-20)}{100}*600\\\\=\frac{80}{100}*600[/tex]

                     = 80 * 6 = $ 480

Cost price for Bob = Selling price of Abel = $ 480

Bob's cost Price = $480

Selling price = [tex]\frac{100+Profit}{100}*CP\\\\[/tex]

                     [tex]= \frac{100+5}{100}*480\\=\frac{105}{100}*480[/tex]

                      = $ 504

Amount paid by Charles =$ 504

A random sample of 13 teenagers were surveyed for a hypothesis test about the mean weekly amount spent on convenience goods. Researchers conduct a one-mean hypothesis test, at the 1% significance level, to test whether the average spent per week on convenience goods is greater than 50 dollars.

Answers

Answer:

Please find the complete question and the graph in the attached file.

Step-by-step explanation:

On the basis of the data,

The level of importance is [tex]\alpha = 0.01[/tex]

Freedom levels [tex]= n -1 = 13 -1 = 12[/tex]

For the right-tailed test, the critical value is [tex]t_c = 2.681[/tex]

(Partially t-table permitted [tex]\alpha = 0.01 \ and\ df =12[/tex])

help me plz i will give you brainlist plzzzzz i beg you​

Answers

Answer:

Solution given:

[tex]\bold{4*3^{x+1}-9^{x}=27}[/tex]

let 3^xbe a

4*a*3-a²=27

a²-12a+27=0

a²-9a-3a+27=0

a(a-9)-3(a-9)=0

(a-9)(a-3)=0

either

a=9

3^x=3²

:.x=2

or

a=3

3^x=3^1

x=1

x=2,1

I need help with this

Answers

Answer:

x = - 4

Step-by-step explanation:

Given a parabola in standard form

f(x) = ax² + bx + c ( a ≠ 0 )

Then the equation of the axis of symmetry is

x = - [tex]\frac{b}{2a}[/tex]

f(x) = 2x² + 16x - 19 ← is in standard form

with a = 2, b = 16

Then the equation of the axis of symmetry is

x = - [tex]\frac{16}{4}[/tex] = - 4

help me with this math question

Answers

Answer: A. In 2015 there will be more seagulls than chickadees.

Step-by-step explanation: if you look at the chart you can see the seagull population keeps growing and the chickadee population keeps decreasing.

Someone help me please

Answers

Answer:

The x intercept is (-7.5,0) and the y-intercept is (0,5.5)

llahkdaclicka. ima answer this

Answers

Answer:

?

Step-by-step explanation:

Answer: Okay………………????????

What is the x-intercept of the line with equation 3y - 8x = 10? Represent your answer as a point in (x, y) form.
The solution is

Answers

Answer:

(-1.25, 0) or (-5/4, 0)

Step-by-step explanation:

The x-intercept is when y = 0, so let's plug 0 into the equation:

3(0) - 8x = 10

Now we use basic algebra to solve for x:

0 - 8x = 10

-8x = 10

x = -5/4 or -1.25

So the answer is (-1.25, 0) or (-5/4, 0).

Hope this helps (●'◡'●)

What x-value are a solution to the system? (select EACH correct answer)​

Answers

Step-by-step explanation:

the answer is correct and helpful

x=1

Mr. Ramadhan wants to save money to buy a house so he puts 21% of his earnings into his savings account. How much money does he save for his house?

Answers

Answer:

79%

Step-by-step explanation:

He has 100% to start off. If he puts 21% into savings you subtract that from the starting amount. 100 - 21 = 79. Therefore the answer is 79%.

Answer

well how much is his earnings?

Step-by-step explanation:

Scarlett tried to shade all the factors of 15 on the hundred chart.What mistake did Scarlett make? A.Scarlett shaded the factors of 5 instead of the factors of 15. B.Scarlett did not make a mistake. C.Scarlett shaded the multiples instead of the factors of 15. D.Scarlett shaded 15, but factors are always less than the number.

Answers

Answer:

C. Scarlett shaded the multiples instead of the factors of 15

Step-by-step explanation:

Scarlett tried to shade all the factors of 15 on the hundred chart 15,45,75,30,60,90 What mistake did she make

She shaded 15,45,75,30,60,90 on the chart

Factors of 15 are 1, 3, 5 and 15

15, 45, 75, 30, 60, 90 are multiples of 15

That is,

15 × 1 = 15

15 × 3 = 45

15 × 5 = 75

15 × 2 = 30

15 × 4 = 60

15 × 6 = 90

Therefore Scarlett's mistake is:

C. Scarlett shaded the multiples instead of the factors of 15

Other Questions
On her summer abroad in France, Jane bought a pair of shoes for 54.82 euros. The store owner only had francs to give her as change. She gave him 55 euros. How much did he give her back in francs SP= 328, Profit = Rs 29 (I GOT AN 80% )1.(D) Ricardo has r minutes left on his cell-phone card. He wants to call his friend and still have 8 minutes left over to make calls later. Which expression describes the length of time he can talk to his friend? D) r 82.(D) Lorraine rode her bike one-sixth the distance Jeremy rode his bike. If b represents the distance Jeremy rode, which expression represents the distance Lorraine rode her bike? D) b --- 63.(A) Raoul rode an elevator up the floors. He entered on the 5'thand exited on the 34'th sloor. Which equation shows an equality between two different ways of expressing the Raoul exited ? A) 34 = 5 + e4.(B) Hank went to an apple orchard and paid an entrance fee of $5 and bought a pounds of apples at $2 per pound. Hank paid a total of $19.Which equation shows an equality between two different ways of expressing how much Hank paid in all? B) 5 + 2a = 195.(A) Chens house is 53 years old. This is 5 more than twice the age of Sunnys house, h.Which equation shows an equality between two different ways of expressing the age of Chens house? A) 53 = 2h + 56.(A) Mandy and four of her friends want to see a movie. If m represents the cost for one person to see a movie, which expression represents the cost for Mandy and her friends to see a movie? A) 5m7.(WRONG ANSWER IS D) Natalie and Juan collect marbles. The number of marbles that Natalie has is 4 more than 3 times the number of marbles Juan has. If m represents the number of marbles Juan has, which of the following expressions represents the number of marbles Natalie has?A) 4m + 3B) 3m + 4C) 3m 4D) DO NOT PICK ITS WRONG 8.(WRONG ANSWER IS D) Hannah baked twice the number of cookies Sam baked. Sam baked 36 cookies. Let c represent the number of cookies Hannah baked.Which equation shows an equality between two different ways of expressing how many cookies Sam baked?A) 36 = c 2B) 36 = c + 2C) 36 = c 2D)DO NOT PICK ITS WRONG9.(B) There were c members in the chess club before 8 more people joined. Which expression describes the number of members in the chess club now? B) c + 810.(C)C) P --- + 1.5 4 (all the correct answers are next to the numbers and it says do not pick which everyone's where wrong which was 7 and 8 i hope this helps you) Set the initial bead height to 3.00 m. Click Play. Notice that the ball makes an entire loop. What is the minimum height required for the ball to make an entire loop who is the first prime minister of India The radius of a sphere is increasing at a rate of 5 mm/s. How fast is the volume increasing (in mm3/s) when the diameter is 40 mm Alex can cut a cord into 7 pieces in 36 seconds. How long will it take him to cut the cord into 12 pieces? what is 2 1/2 divided by 1/3 {pls hurry the teacher is not letting us use brainly} -0.6the decimal in equivalent fraction form James and his family spend a morning picking peaches. They fill several bags. This line plot shows the weight of each bag. How many more pounds does one of the heaviest bags weigh than one of the lightest bags? Enter your answer as a fraction in simplest form by filling in the boxes. A basketball is shot by a player at a height of 2.0 m. The initial anglewas 53 above horizontal. At the highest point the ball was travelling 6m/s. If he scored (the ball went through the rim that is 3.00 m above theground), what was the players horizontal distance from the basket? I will literally give you my life if you. help me with these integer questions- PLEASE HELP WILL MARK BRAINLIEST! Also please explain the answer Mary has the opportunity to attend the New York Metropolitan Opera or a New York Mets game. The travel agent is explaining the pros and cons of both. What type of listening will Mary most likely employ?a) deliberativeb) critical please me in the math[tex]6 {x}^{6} + 6 {x}^{4} + 6 {x}^{2} and \: \\ 4 {x}^{6} - 4 {x}^{x} \\ it \: is \: lcm[/tex] Which answer explains the correct way to move the decimal to find the quotient of 14.6 10,000?three places to the right.three places to the left.four places to the left.four places to the right A macaroni and cheese recipe calls for 's of a 2 % pound block of cheese. How many poundsare needed?Mr. Williams ask to buy 1/2 of a pan of brownies that is 2/3 full. What fraction of the original pan does he buy? Rational between 16 and 17 Explain the relevance of psychology The product of two numbers is 60 and thei r sum is it, find the Numbers